Finding a useful denial of a injective function and a surjective function

Click For Summary
The discussion focuses on finding the useful denials of injective and surjective functions. For an injective function, the denial is correctly stated as (∃x1,x2 ∈ X)(x1 ≠ x2 ∧ f(x1) = f(x2)), indicating that distinct inputs can produce the same output. For a surjective function, the denial is (∃y ∈ Y)(∀x ∈ X)(y ≠ f(x)), meaning there exists an output in the codomain that is not produced by any input. Both proposed denials are confirmed to be correct. The conversation emphasizes understanding the logical structure of these mathematical definitions.
ver_mathstats
Messages
258
Reaction score
21

Homework Statement


Find the useful denial of a injective function and a surjective function.

Homework Equations

The Attempt at a Solution


I know a one to one function is (∀x1,x2 ∈ X)(x1≠x2 ⇒ f(x1) ≠ f(x2)). So would the useful denial be (∃x1,x2 ∈ X)(x1 ≠ x2 ∧ f(x1) = f(x2))?

I know a onto function is (∀y ∈ Y)(∃x ∈ X)(y=f(x)). So would the useful denial be (∃y ∈ Y)(∀x ∈ X)(y≠f(x))?

Thank you.
 
Physics news on Phys.org
ver_mathstats said:

Homework Statement


Find the useful denial of a injective function and a surjective function.

Homework Equations

The Attempt at a Solution


I know a one to one function is (∀x1,x2 ∈ X)(x1≠x2 ⇒ f(x1) ≠ f(x2)). So would the useful denial be (∃x1,x2 ∈ X)(x1 ≠ x2 ∧ f(x1) = f(x2))?

I know a onto function is (∀y ∈ Y)(∃x ∈ X)(y=f(x)). So would the useful denial be (∃y ∈ Y)(∀x ∈ X)(y≠f(x))?

Thank you.
Both is correct.
 
fresh_42 said:
Both is correct.
Thank you.
 
Injective means
upload_2019-2-22_20-29-14.png

cannot happen, and surjective means
upload_2019-2-22_20-30-40.png


##y## cannot exist.
 

Attachments

  • upload_2019-2-22_20-29-14.png
    upload_2019-2-22_20-29-14.png
    356 bytes · Views: 402
  • upload_2019-2-22_20-30-40.png
    upload_2019-2-22_20-30-40.png
    1.2 KB · Views: 403
Question: A clock's minute hand has length 4 and its hour hand has length 3. What is the distance between the tips at the moment when it is increasing most rapidly?(Putnam Exam Question) Answer: Making assumption that both the hands moves at constant angular velocities, the answer is ## \sqrt{7} .## But don't you think this assumption is somewhat doubtful and wrong?

Similar threads

Replies
30
Views
3K
  • · Replies 26 ·
Replies
26
Views
3K
  • · Replies 5 ·
Replies
5
Views
3K
  • · Replies 14 ·
Replies
14
Views
3K
  • · Replies 9 ·
Replies
9
Views
5K
  • · Replies 8 ·
Replies
8
Views
2K
Replies
8
Views
2K
  • · Replies 2 ·
Replies
2
Views
1K
  • · Replies 1 ·
Replies
1
Views
2K
  • · Replies 9 ·
Replies
9
Views
2K